NAPLEX Practice Questions | Free 200-Question Bank & Mock Exams

NAPLEX Practice Questions & Question Bank

Your one-stop resource for passing the North American Pharmacist Licensure Examination. Access 200+ high-yield clinical questions, full-length simulations, and domain-specific practice modules.

📚 NAPLEX Practice Questions (200 Questions)

1. A 63-year-old man with a history of ischemic cardiomyopathy, hypertension, type 2 diabetes, and stage 3 chronic kidney disease presents to clinic reporting progressive exertional dyspnea and difficulty lying flat without pillows. His medications include lisinopril 20 mg daily, furosemide 40 mg twice daily, metformin 1000 mg twice daily, and aspirin 81 mg daily. Vital signs: BP 108/66 mmHg, HR 74 bpm. Physical exam shows mild bilateral leg edema. Recent echocardiogram shows EF 30 percent. Labs: K⁺ 4.5 mEq/L, eGFR 42 mL/min. According to guideline-directed therapy, which medication should be added to reduce mortality?

  • A. Amlodipine
  • B. Carvedilol
  • C. Digoxin
  • D. Verapamil

Correct Answer: B. Carvedilol

2. A 58-year-old woman presents to the emergency department with a 4-day history of fever, purulent sputum, pleuritic chest pain, and shortness of breath. Past history includes COPD (moderate), hypertension, and a 20-pack-year smoking history. She reports an anaphylactic reaction to penicillin during childhood. Vitals: T 38.3°C, HR 118, RR 26, BP 108/68, SpO₂ 89 percent on room air. Chest X-ray shows left lower lobe consolidation. Sputum culture reveals gram-positive diplococci. WBC 15,200/mm³. CrCl 76 mL/min. Which empiric antibiotic regimen is MOST appropriate?

  • A. Ceftriaxone plus azithromycin
  • B. Amoxicillin/clavulanate
  • C. Levofloxacin monotherapy
  • D. Piperacillin/tazobactam

Correct Answer: C. Levofloxacin monotherapy

3. A 75-year-old man with a history of type 2 diabetes, hypertension, and chronic kidney disease presents with confusion and generalized weakness. His daughter reports vomiting and diarrhea for the last 2 days with poor oral intake. Medications include metformin 1000 mg BID, lisinopril 10 mg daily, and glipizide 5 mg daily. Vitals: BP 92/54, HR 112, RR 22. Labs: pH 7.25, lactate 6.1 mmol/L, SCr 3.5 mg/dL (baseline 1.9), HCO₃⁻ 15 mEq/L, anion gap 18. Chest X-ray normal. Which medication most likely contributed to this patient’s condition?

  • A. Lisinopril
  • B. Metformin
  • C. Glipizide
  • D. Simvastatin

Correct Answer: B. Metformin

4. A 49-year-old woman underwent abdominal surgery 2 days ago and has been receiving IV hydromorphone via PCA pump for postoperative pain. Overnight, she also received diphenhydramine for itching and lorazepam for anxiety. This morning she is difficult to arouse, RR 8 breaths/min, O₂ saturation 87 percent on 2 L nasal cannula. Blood pressure is stable. Which action should the pharmacist recommend FIRST?

  • A. Reduce PCA dose
  • B. Administer naloxone
  • C. Obtain arterial blood gas
  • D. Discontinue diphenhydramine

Correct Answer: B. Administer naloxone

5. A 57-year-old man presents for evaluation of uncontrolled hypertension. He is currently taking lisinopril 40 mg daily and amlodipine 10 mg daily, with documented adherence. BP today is 164/98 mmHg. He also reports chronic back pain for which he takes ibuprofen 600 mg three times daily. Labs: K⁺ 4.3 mEq/L, eGFR 82 mL/min, microalbuminuria present. Which intervention is MOST appropriate?

  • A. Add hydrochlorothiazide
  • B. Add spironolactone
  • C. Discontinue ibuprofen
  • D. Switch lisinopril to losartan

Correct Answer: C. Discontinue ibuprofen

6. A 71-year-old woman with a history of atrial fibrillation presents for anticoagulation assessment. CHADS₂-VASc score is 5. She has normal renal and hepatic function. She takes diltiazem, hydrochlorothiazide, and metformin. She reports no bleeding history. Her weight is 66 kg and age 71 years. Labs: SCr 0.9 mg/dL. Which anticoagulant regimen is most appropriate?

  • A. Apixaban 2.5 mg BID
  • B. Apixaban 5 mg BID
  • C. Rivaroxaban 15 mg daily
  • D. Warfarin target INR 1–2

Correct Answer: B. Apixaban 5 mg BID

7. A 59-year-old man presents with worsening shortness of breath. He has COPD and uses tiotropium daily and albuterol as needed. Over the past 3 months, he has had two exacerbations requiring oral steroids. He continues to smoke ½ pack per day. Vitals: RR 22, SpO₂ 92 percent. Lung exam shows diffuse wheezing. Which medication should be added to reduce exacerbation frequency?

  • A. Nebulized ipratropium
  • B. ICS/LABA combination
  • C. Theophylline
  • D. Roflumilast monotherapy

Correct Answer: B. ICS/LABA combination

8. A 67-year-old woman is being treated for type 2 diabetes. Current therapy includes metformin 1000 mg BID and glimepiride 8 mg daily. A1C is 9.1 percent. She has a history of ASCVD. Weight 94 kg, BMI 32. Labs: eGFR 60 mL/min. According to ADA guidelines, which medication should be added?

  • A. Sitagliptin
  • B. Liraglutide
  • C. Pioglitazone
  • D. Acarbose

Correct Answer: B. Liraglutide

9. A 74-year-old man reports persistent leg swelling and discomfort. He was recently started on amlodipine 10 mg daily for hypertension. He denies chest pain. BP 132/74 mmHg, HR 68. Physical exam reveals bilateral pitting edema. Labs are normal. Which intervention is MOST appropriate?

  • A. Add furosemide 20 mg daily
  • B. Switch amlodipine to lisinopril
  • C. Add spironolactone
  • D. Increase amlodipine dose

Correct Answer: B. Switch amlodipine to lisinopril

10. A 45-year-old man with severe asthma presents to clinic. Current meds: albuterol (daily use), fluticasone/salmeterol 250/50, montelukast 10 mg nightly. Over the last month he has 3 nighttime awakenings weekly and required prednisone once. Vitals: RR 20, SpO₂ 96 percent. Which therapy should be added next?

  • A. Increase ICS/LABA dose
  • B. Add tiotropium
  • C. Add prednisone daily
  • D. Switch to SABA-only

Correct Answer: A. Increase ICS/LABA dose

11. A 69-year-old woman presents with acute confusion and hallucinations. She was recently prescribed ciprofloxacin for a urinary tract infection. Her history includes dementia, CKD stage 3, and hypertension. Vitals stable. Labs: eGFR 42 mL/min. UA improving. Which is the most likely cause of her symptoms?

  • A. Progression of dementia
  • B. Ciprofloxacin-induced neurotoxicity
  • C. Hyponatremia
  • D. Uremic encephalopathy

Correct Answer: B. Ciprofloxacin-induced neurotoxicity

12. A 62-year-old man with diabetes presents for insulin adjustment. Current therapy: glargine 30 units nightly and lispro 6 units before meals. His fasting glucose values are 170–190 mg/dL consistently. A1C is 8.9 percent. Which adjustment is most appropriate?

  • A. Increase glargine by 2 units every 3–4 days
  • B. Increase mealtime lispro to 12 units
  • C. Add pioglitazone
  • D. Switch glargine to NPH

Correct Answer: A. Increase glargine by 2 units every 3–4 days

13. A 55-year-old man with cirrhosis presents with confusion. Physical exam shows asterixis. Labs: ammonia elevated. He has been adherent to furosemide and spironolactone for ascites. No evidence of GI bleed. Which medication is FIRST-line treatment?

  • A. Rifaximin alone
  • B. Lactulose
  • C. Metronidazole
  • D. Midodrine

Correct Answer: B. Lactulose

14. A 41-year-old woman with a long history of epilepsy presents for therapy review. She reports gum overgrowth and recent imbalance. Current medication: phenytoin. Phenytoin level is 8 mcg/mL (subtherapeutic). She states she has been taking her medication correctly. Which factor MOST likely affects her levels?

  • A. High albumin levels
  • B. Nonlinear pharmacokinetics of phenytoin
  • C. Phenytoin’s renal clearance
  • D. Phenytoin’s short half-life

Correct Answer: B. Nonlinear pharmacokinetics of phenytoin

15. A 29-year-old woman presents with dysuria, urinary frequency, and suprapubic pain for 2 days. UA positive for leukocyte esterase and nitrites. She is not pregnant and has no drug allergies. Which is the most appropriate treatment?

  • A. Nitrofurantoin 100 mg BID for 5 days
  • B. Ciprofloxacin 500 mg BID for 14 days
  • C. Linezolid for 3 days
  • D. IV ceftriaxone

Correct Answer: A. Nitrofurantoin 100 mg BID for 5 days

16. A 78-year-old man is admitted with pneumonia and started on vancomycin. Weight: 82 kg, CrCl 35 mL/min. Serum creatinine stable. Which dosing regimen is MOST appropriate?

  • A. 1 g q12h
  • B. 1 g q24h
  • C. 1.5 g q12h
  • D. 750 mg q24h

Correct Answer: B. 1 g q24h

17. A 60-year-old man taking warfarin is prescribed trimethoprim/sulfamethoxazole for a skin infection. Two days later, INR is 5.6 and he has no bleeding. What is the best management?

  • A. Hold warfarin 1–2 doses
  • B. Give oral vitamin K 5 mg
  • C. Give IV vitamin K
  • D. Continue warfarin

Correct Answer: A. Hold warfarin 1–2 doses

18. A 48-year-old man on clozapine develops fever and sore throat. CBC shows WBC 1,400/mm³ and ANC 800/mm³. What is the most appropriate action?

  • A. Continue therapy
  • B. Reduce dose
  • C. Discontinue clozapine
  • D. Add lithium

Correct Answer: C. Discontinue clozapine

19. A 72-year-old woman with depression is switching from fluoxetine to phenelzine. Which recommendation is correct?

  • A. Switch immediately
  • B. Wait 2 days
  • C. Wait 7 days
  • D. Wait 5 weeks

Correct Answer: D. Wait 5 weeks

20. A 64-year-old woman has osteoporosis diagnosed on DEXA scan. She has difficulty remaining upright for 30 minutes due to back pain. Which medication is MOST appropriate?

  • A. Alendronate
  • B. Risedronate
  • C. Denosumab
  • D. Ibandronate oral

Correct Answer: C. Denosumab

21. A 72-year-old man with atrial fibrillation, hypertension, and chronic heart failure presents for anticoagulation adjustment. He has been taking warfarin 5 mg daily for 6 months with stable INR values between 2.0–2.6. His new medications include amiodarone, started 5 days ago for rhythm control. Today his INR is 4.9. He denies bleeding. Labs: SCr 1.0 mg/dL, liver function normal. Which intervention is MOST appropriate?

  • A. Continue warfarin at current dose
  • B. Hold 1–2 warfarin doses and reduce maintenance dose
  • C. Administer oral vitamin K 5 mg
  • D. Switch to aspirin 81 mg daily

Correct Answer: B. Hold 1–2 warfarin doses and reduce maintenance dose

22. A 61-year-old woman presents with severe headache, photophobia, and nausea. She has a history of migraine with aura. She reports taking sumatriptan 100 mg twice in the past 24 hours with little relief. Her medications include sertraline for depression and propranolol for prevention. Vital signs are normal. Which medication is CONTRAINDICATED for acute migraine treatment in this patient due to risk of serotonin syndrome?

  • A. Rizatriptan
  • B. Metoclopramide
  • C. Ketorolac
  • D. Dexamethasone

Correct Answer: A. Rizatriptan

23. A 69-year-old man is admitted for community-acquired pneumonia. He is started on ceftriaxone and azithromycin. On day 2, sputum culture reveals methicillin-resistant Staphylococcus aureus (MRSA). Vitals stable. He has a history of renal insufficiency (CrCl 48 mL/min). Which antibiotic is MOST appropriate to add?

  • A. Vancomycin
  • B. Cefepime
  • C. Linezolid
  • D. Daptomycin

Correct Answer: A. Vancomycin

24. A 58-year-old woman presents with worsening lower-extremity edema, abdominal distension, and fatigue. She has cirrhosis from chronic hepatitis C. Medications include furosemide and lactulose. She has not been taking spironolactone for 2 months because of breast tenderness. Vitals: BP 104/64, HR 82. Exam reveals shifting dullness and 3+ edema. Labs: Na⁺ 130 mEq/L, K⁺ 3.4 mEq/L, SCr 0.9 mg/dL. Which intervention is MOST appropriate?

  • A. Restart spironolactone at a lower dose
  • B. Increase furosemide alone
  • C. Add hydrochlorothiazide
  • D. Switch lactulose to rifaximin

Correct Answer: A. Restart spironolactone at a lower dose

25. A 51-year-old woman with uncontrolled type 2 diabetes presents for follow-up. Current therapy: metformin 1000 mg BID, dapagliflozin 10 mg daily, and basal insulin glargine 34 units daily. Fasting glucose remains 160–180 mg/dL. Weight 88 kg, eGFR 58 mL/min. A1C is 9.2 percent. Which is the BEST next step?

  • A. Increase basal insulin by 2 units every 3 days
  • B. Add mealtime rapid-acting insulin
  • C. Switch dapagliflozin to pioglitazone
  • D. Increase metformin dose

Correct Answer: B. Add mealtime rapid-acting insulin

26. A 77-year-old man presents with worsening confusion. He was recently prescribed diphenhydramine for insomnia. Past medical history includes BPH, dementia, and hypertension. Vitals normal. Labs normal. Which medication change is MOST appropriate?

  • A. Increase diphenhydramine dose
  • B. Switch diphenhydramine to melatonin
  • C. Add risperidone
  • D. Add lorazepam at bedtime

Correct Answer: B. Switch diphenhydramine to melatonin

27. A 33-year-old man presents with fever, chills, vomiting, and left flank pain for 24 hours. UA shows WBC clumps and positive leukocyte esterase. CT reveals left-sided pyelonephritis. He has no allergies. Vitals stable. CrCl 90 mL/min. Which is the most appropriate INITIAL therapy?

  • A. Nitrofurantoin
  • B. Ciprofloxacin
  • C. Doxycycline
  • D. Metronidazole

Correct Answer: B. Ciprofloxacin

28. A 48-year-old man presents with new-onset severe back pain after lifting heavy boxes at work. He has hypertension on lisinopril. He requests something “stronger than ibuprofen.” His BP is 126/70. No neurologic deficits present. Which therapy is MOST appropriate?

  • A. Oxycodone
  • B. Cyclobenzaprine
  • C. Ketorolac IM
  • D. Tramadol

Correct Answer: B. Cyclobenzaprine

29. A 57-year-old woman presents with increasing shortness of breath and orthopnea. She is on lisinopril, carvedilol, furosemide, and rosuvastatin. Her EF is 28 percent. Labs show potassium 4.3 mEq/L and SCr 1.1 mg/dL. She is not on an SGLT2 inhibitor. Which medication should be added to reduce heart failure hospitalizations and mortality?

  • A. Spironolactone
  • B. Dapagliflozin
  • C. Isosorbide mononitrate
  • D. Amlodipine

Correct Answer: B. Dapagliflozin

30. A 61-year-old man with PTSD and alcohol use disorder presents with confusion and ataxia. His girlfriend reports poor diet and daily alcohol intake. Vitals stable. Labs normal. Which supplementation is MOST appropriate?

  • A. Folic acid
  • B. Thiamine
  • C. Vitamin E
  • D. Riboflavin

Correct Answer: B. Thiamine

31. A 70-year-old woman presents with difficulty breathing and wheezing. She uses albuterol daily and fluticasone/salmeterol 250/50 twice daily. Over the last month she has had nighttime symptoms several times per week. Vitals: RR 24, SpO₂ 95 percent. What is the next step in therapy?

  • A. Switch to SABA-only
  • B. Increase ICS/LABA dose
  • C. Add theophylline
  • D. Add oral steroids permanently

Correct Answer: B. Increase ICS/LABA dose

32. A 67-year-old man with end-stage renal disease on hemodialysis presents with muscle cramps, bone pain, and pruritus. Labs: Ca²⁺ 7.9 mg/dL, phosphate 7.2 mg/dL, PTH markedly elevated. Which medication is MOST appropriate?

  • A. Cinacalcet
  • B. Vitamin D only
  • C. Calcium chloride IV
  • D. Loop diuretic

Correct Answer: A. Cinacalcet

33. A 52-year-old man presents with severe abdominal pain radiating to the back. He drinks heavily and has nausea and vomiting. Vitals: BP 100/60, HR 110. Labs: lipase markedly elevated, Ca²⁺ 7.8 mg/dL. CT confirms acute pancreatitis. Which management is MOST appropriate initially?

  • A. Initiate early enteral feeding
  • B. Start broad-spectrum antibiotics
  • C. Administer opioid analgesia and IV fluids
  • D. Start TPN immediately

Correct Answer: C. Administer opioid analgesia and IV fluids

34. A 59-year-old woman comes for medication review. She is taking simvastatin 40 mg nightly and recently started clarithromycin for sinusitis. Today she reports muscle soreness. Labs pending. Which action is MOST appropriate?

  • A. Continue both meds
  • B. Discontinue simvastatin
  • C. Reduce simvastatin to 10 mg
  • D. Increase simvastatin dose

Correct Answer: B. Discontinue simvastatin

35. A 45-year-old woman complains of worsening acid reflux despite using omeprazole 20 mg daily for 6 months. She takes ibuprofen daily and drinks two cups of coffee each morning. She denies alarm symptoms. Which intervention is MOST appropriate?

  • A. Increase omeprazole to 40 mg daily
  • B. Add sucralfate
  • C. Add ranitidine
  • D. Schedule upper endoscopy

Correct Answer: A. Increase omeprazole to 40 mg daily

36. A 72-year-old man requires warfarin therapy but refuses frequent INR checks. He has stable renal function (CrCl 55 mL/min). He is not on interacting meds. BMI is 26. Which anticoagulant is MOST appropriate?

  • A. Apixaban
  • B. Dabigatran
  • C. Enoxaparin
  • D. Heparin

Correct Answer: A. Apixaban

37. A 28-year-old woman presents to the ED with palpitations and tachycardia. She recently increased her levothyroxine dose independently after reading online advice. Labs: TSH suppressed, free T4 elevated. Vitals: HR 126. Which intervention is MOST appropriate?

  • A. Increase levothyroxine
  • B. Continue levothyroxine
  • C. Reduce levothyroxine dose
  • D. Switch to liothyronine

Correct Answer: C. Reduce levothyroxine dose

38. A 63-year-old man presents with acute urinary retention. He has BPH and is taking diphenhydramine nightly for sleep. Bladder scan shows 800 mL retained urine. Vitals normal. Which medication contributed to this condition?

  • A. Diphenhydramine
  • B. Lisinopril
  • C. Metformin
  • D. Atorvastatin

Correct Answer: A. Diphenhydramine

39. A 72-year-old woman on chronic prednisone presents with new onset hip pain. DEXA shows reduced bone mass. Which medication should be initiated?

  • A. Calcium alone
  • B. Alendronate
  • C. Estrogen
  • D. Teriparatide only

Correct Answer: B. Alendronate

40. A 57-year-old man presents with chronic pain and is requesting opioid therapy. Review of PDMP reveals he received oxycodone from two other providers last month. What is the MOST appropriate action?

  • A. Provide a month of oxycodone
  • B. Deny care permanently
  • C. Discuss findings with the patient and coordinate care
  • D. Report immediately to law enforcement

Correct Answer: C. Discuss findings with the patient and coordinate care

41. A 76-year-old woman with HFpEF, hypertension, osteoarthritis, and chronic kidney disease presents with worsening shortness of breath. She has been taking ibuprofen 600 mg TID for joint pain over the past month. Today her BP is 162/88 mmHg, HR 78 bpm, weight increased by 2.5 kg from baseline, and crackles are heard at lung bases. Labs: SCr 1.9 mg/dL (baseline 1.3), BNP elevated. Which intervention is MOST appropriate?

  • A. Increase ibuprofen dose
  • B. Discontinue ibuprofen
  • C. Add spironolactone
  • D. Add hydralazine

Correct Answer: B. Discontinue ibuprofen

42. A 62-year-old man with a history of alcohol use disorder presents with tremors, sweating, anxiety, and palpitations. Symptoms began 12 hours after his last drink. BP 158/96 mmHg, HR 120 bpm, RR 22. He takes no prescription medications. Labs: glucose 98 mg/dL, electrolytes normal. Which medication is FIRST-line therapy?

  • A. Haloperidol
  • B. Lorazepam
  • C. Chlordiazepoxide
  • D. Methadone

Correct Answer: C. Chlordiazepoxide

43. A 54-year-old woman presents with severe acute-onset left lower quadrant abdominal pain, nausea, and fever. Past history: diverticulosis, hypertension, hyperlipidemia. Vitals: T 38.2°C, HR 104 bpm. CT scan confirms acute diverticulitis with no perforation. She is stable and tolerating oral intake. Which is the MOST appropriate outpatient antibiotic regimen?

  • A. Ciprofloxacin plus metronidazole
  • B. Nitrofurantoin monotherapy
  • C. Daptomycin
  • D. Linezolid monotherapy

Correct Answer: A. Ciprofloxacin plus metronidazole

44. A 69-year-old man with Parkinson disease presents with hallucinations and confusion. He is currently taking carbidopa/levodopa, pramipexole, and entacapone. Vitals stable. His daughter reports he recently started taking diphenhydramine for sleep. Labs normal. Which intervention is MOST appropriate?

  • A. Increase pramipexole
  • B. Add haloperidol
  • C. Discontinue diphenhydramine
  • D. Start risperidone

Correct Answer: C. Discontinue diphenhydramine

45. A 60-year-old man with severe COPD presents after a new episode of dyspnea and coughing. He is on tiotropium daily, salmeterol twice daily, and albuterol PRN. He has had 2 exacerbations in 6 months requiring steroids. He also reports fatigue and dark stools. Vitals: BP 92/58, HR 112, RR 26. Labs: Hb 8.2 g/dL, ferritin low. Which intervention is MOST appropriate?

  • A. Start ICS/LABA
  • B. Long-term oral steroids
  • C. Evaluate for GI bleed
  • D. Increase tiotropium dose

Correct Answer: C. Evaluate for GI bleed

46. A 40-year-old man presents with severe abdominal pain, vomiting, and a history of long-term NSAID use for back pain. Vitals: BP 98/62, HR 110. Labs: Hgb 9.4 g/dL. Stool guaiac positive. Endoscopy reveals a gastric ulcer. He is currently taking ibuprofen 600 mg TID. Which medication should be recommended?

  • A. Continue ibuprofen with food
  • B. Start high-dose PPI therapy
  • C. Add misoprostol only
  • D. Start oral steroids

Correct Answer: B. Start high-dose PPI therapy

47. A 72-year-old man on chronic warfarin is started on amiodarone for arrhythmia control. Five days later his INR rises from 2.4 to 5.1. He is asymptomatic. Labs normal. Which management is most appropriate?

  • A. Continue warfarin dose
  • B. Hold 1–2 doses and reduce warfarin maintenance dose
  • C. Give IV vitamin K
  • D. Switch to aspirin

Correct Answer: B. Hold 1–2 doses and reduce warfarin maintenance dose

48. A 44-year-old woman presents with recurrent sinus infections, chronic cough, and fatigue. She has been using multiple OTC cold remedies, including pseudoephedrine, loratadine, and dextromethorphan. Vitals: BP 162/94 mmHg, HR 104 bpm. She denies pain. Which medication MOST likely contributed to her elevated BP?

  • A. Dextromethorphan
  • B. Loratadine
  • C. Pseudoephedrine
  • D. Guaifenesin

Correct Answer: C. Pseudoephedrine

49. A 38-year-old woman with bipolar disorder is in the ED for tremor, diarrhea, confusion, and vomiting. She has been taking lithium for years. Labs: Na⁺ 146 mEq/L, SCr 2.8 mg/dL (baseline 1.0), lithium level 2.0 mEq/L. Which intervention is MOST appropriate?

  • A. Increase lithium
  • B. Switch to lamotrigine
  • C. Hold lithium and provide IV fluids
  • D. Give flumazenil

Correct Answer: C. Hold lithium and provide IV fluids

50. A 68-year-old man with congestive heart failure and chronic kidney disease presents with muscle weakness. He is on furosemide 80 mg BID and digoxin 0.25 mg daily. Labs: K⁺ 2.7 mEq/L, digoxin level 2.7 ng/mL. EKG shows frequent PVCs. Which is the MOST appropriate intervention?

  • A. Increase digoxin
  • B. Administer IV potassium and hold digoxin
  • C. Administer IV calcium
  • D. Add spironolactone

Correct Answer: B. Administer IV potassium and hold digoxin

51. A 57-year-old man presents with burning epigastric pain after meals. He has been self-medicating with antacids and ibuprofen. Vitals stable. Labs: H. pylori positive. Which regimen is MOST appropriate?

  • A. Clarithromycin, amoxicillin, and PPI
  • B. Metronidazole only
  • C. Ciprofloxacin only
  • D. PPI monotherapy

Correct Answer: A. Clarithromycin, amoxicillin, and PPI

52. A 64-year-old woman presents with worsening anxiety. She has been taking alprazolam 1 mg four times daily for 2 years. She feels the medication “isn’t working anymore.” She asks for a higher dose. Which is the MOST appropriate response?

  • A. Increase alprazolam dose
  • B. Transition to a gradual taper schedule
  • C. Switch to diazepam immediately
  • D. Add hydromorphone at bedtime

Correct Answer: B. Transition to a gradual taper schedule

53. A 37-year-old man with HIV on efavirenz/emtricitabine/tenofovir presents with vivid dreams, dizziness, and mood changes. These symptoms started after beginning therapy. Vitals normal. Labs stable. Which medication is MOST likely responsible?

  • A. Efavirenz
  • B. Emtricitabine
  • C. Tenofovir
  • D. Ritonavir

Correct Answer: A. Efavirenz

54. A 55-year-old man with atrial fibrillation presents with difficulty breathing after starting sotalol 3 days ago. Vitals: HR 52, BP 90/60, QTc prolonged on ECG. Which is the MOST appropriate intervention?

  • A. Increase sotalol
  • B. Switch to amiodarone
  • C. Stop sotalol and monitor ECG
  • D. Continue sotalol

Correct Answer: C. Stop sotalol and monitor ECG

55. A 29-year-old woman presents with recurrent yeast infections. She is on long-term prednisone for lupus. She requests a medication to take “whenever she gets symptoms.” Which is MOST appropriate?

  • A. Fluconazole 150 mg single dose
  • B. Amoxicillin
  • C. Acyclovir
  • D. Metronidazole

Correct Answer: A. Fluconazole 150 mg single dose

56. A 78-year-old woman with chronic heart failure is admitted for worsening edema. She is receiving IV furosemide. After 24 hours, urine output is low and weight unchanged. Labs: Na⁺ 132 mEq/L, K⁺ 4.3 mEq/L, SCr 1.9 mg/dL. She appears volume overloaded. Which intervention is MOST appropriate?

  • A. Add metolazone
  • B. Reduce furosemide
  • C. Give IV fluids
  • D. Add digoxin

Correct Answer: A. Add metolazone

57. A 46-year-old man presents with joint pain and swelling in his right big toe. He has a history of gout. Vitals stable. Labs: uric acid markedly elevated. Which medication is MOST appropriate for acute flare treatment?

  • A. Allopurinol
  • B. Colchicine
  • C. Probenecid
  • D. Febuxostat

Correct Answer: B. Colchicine

58. A 63-year-old man presents after a syncopal episode. He recently started using tadalafil for erectile dysfunction while on his chronic medications, which include sublingual nitroglycerin. Vitals: BP 82/48, HR 98. Which is the MOST likely cause?

  • A. Bradycardia
  • B. Orthostatic hypotension
  • C. Nitrate–PDE-5 inhibitor interaction
  • D. Acute heart failure

Correct Answer: C. Nitrate–PDE-5 inhibitor interaction

59. A 41-year-old woman with rheumatoid arthritis has been on methotrexate for 8 months. She reports fatigue and pallor. Labs: macrocytic anemia. Which supplementation is required?

  • A. Vitamin B12
  • B. Folic acid
  • C. Iron
  • D. Vitamin D

Correct Answer: B. Folic acid

60. A 69-year-old woman with severe asthma presents after an exacerbation. She is discharged with a high-dose ICS/LABA. Which counseling point is MOST important?

  • A. Do not rinse mouth after use
  • B. Stop SABA immediately
  • C. Rinse mouth after each use to prevent thrush
  • D. Store inhaler at freezing temperatures

Correct Answer: C. Rinse mouth after each use to prevent thrush

61. A 58-year-old man with type 2 diabetes, hypertension, and obesity presents for follow-up. Medications: metformin 1000 mg BID, empagliflozin 10 mg daily, lisinopril 20 mg daily, and atorvastatin 40 mg daily. A1C is 8.7 percent. He reports eating irregular meals and skipping breakfast. Fasting BG values are 130–150 mg/dL, but postprandial values are often >250 mg/dL. He does not want insulin yet. eGFR is 75 mL/min. Which medication is MOST appropriate to add?

  • A. Glipizide
  • B. Sitagliptin
  • C. Acarbose
  • D. Regular insulin

Correct Answer: B. Sitagliptin

62. A 63-year-old woman presents with acute onset of unilateral facial drooping, arm weakness, and difficulty speaking that started 45 minutes ago. CT of the head shows no intracranial hemorrhage. She has no history of recent surgery or anticoagulant use. BP is 168/92 mmHg, HR 88 bpm, glucose 124 mg/dL. Which therapy is MOST appropriate at this time?

  • A. Aspirin 81 mg only
  • B. Alteplase IV thrombolysis
  • C. Clopidogrel loading dose only
  • D. Warfarin loading dose

Correct Answer: B. Alteplase IV thrombolysis

63. A 71-year-old man is admitted with urosepsis and receives broad-spectrum antibiotics including vancomycin and piperacillin/tazobactam. After 5 days, his SCr increases from 1.0 mg/dL to 2.6 mg/dL. Urine output has declined. Vancomycin trough is 27 mcg/mL. Which intervention is MOST appropriate?

  • A. Continue both antibiotics
  • B. Discontinue vancomycin and adjust therapy
  • C. Increase vancomycin dose
  • D. Add gentamicin

Correct Answer: B. Discontinue vancomycin and adjust therapy

64. A 46-year-old woman with a history of generalized anxiety disorder has been using alprazolam 0.5 mg three times daily for the past 9 months. She wants to stop “all at once” because she feels better. She denies alcohol or drug use. Which counseling point is MOST appropriate?

  • A. Abrupt discontinuation is safe
  • B. Abrupt discontinuation may cause withdrawal and seizures; gradual taper is needed
  • C. Increase dose briefly before stopping
  • D. Switch immediately to zolpidem

Correct Answer: B. Abrupt discontinuation may cause withdrawal and seizures; gradual taper is needed

65. A 59-year-old man with hypertension and dyslipidemia presents with persistent muscle pain and weakness in his thighs. He has been taking simvastatin 40 mg nightly for 8 months. CK is 1900 IU/L (normal <200). SCr is 1.0 mg/dL. Which intervention is MOST appropriate?

  • A. Continue simvastatin and monitor
  • B. Stop simvastatin and evaluate for rhabdomyolysis
  • C. Reduce simvastatin to 10 mg
  • D. Switch to higher-intensity statin

Correct Answer: B. Stop simvastatin and evaluate for rhabdomyolysis

66. A 68-year-old woman with atrial fibrillation is started on dabigatran for stroke prevention. Her CrCl is 35 mL/min. She asks about handling and storage. Which counseling point is CORRECT?

  • A. Open capsules and sprinkle on food
  • B. Store in original container and use within 4 months
  • C. Crush tablets and mix with water
  • D. Keep in pill organizer for 6 months

Correct Answer: B. Store in original container and use within 4 months

67. A 52-year-old pharmacist is reviewing a new chemotherapy order for doxorubicin. The order appears to exceed the recommended lifetime cumulative dose based on previous cycles. The oncologist is unavailable and the nursing staff is pressuring to prepare the dose quickly due to scheduling. Which is the MOST appropriate action?

  • A. Prepare and dispense the dose as written
  • B. Reduce the dose by half without contacting the physician
  • C. Hold the order and clarify with the prescriber or covering oncologist
  • D. Ask the nurse to decide whether to give the dose

Correct Answer: C. Hold the order and clarify with the prescriber or covering oncologist

68. A 70-year-old man with benign prostatic hyperplasia (BPH), glaucoma, and hypertension presents with worsening urinary retention and blurred vision. He recently started an OTC cold medication recommended by a friend. Medications prior to this included tamsulosin, lisinopril, and latanoprost eye drops. Which component of OTC therapy MOST likely caused his symptoms?

  • A. Guaifenesin
  • B. Dextromethorphan
  • C. Diphenhydramine
  • D. Acetaminophen

Correct Answer: C. Diphenhydramine

69. A 48-year-old woman with Crohn disease is receiving infliximab. Before initiating therapy, she had a negative PPD test and chest X-ray. She now presents with fever, cough, and weight loss after recent travel. Chest imaging reveals cavitary lesions. Which is the MOST important next step?

  • A. Increase infliximab dose
  • B. Test for tuberculosis reactivation
  • C. Start empiric fluoroquinolone
  • D. Start oral prednisone

Correct Answer: B. Test for tuberculosis reactivation

70. A 66-year-old man with diabetes and chronic kidney disease presents with severe hypoglycemia (BG 38 mg/dL) after taking his usual glyburide dose but skipping lunch. He is treated and stabilized in the ED. Which is the MOST appropriate change to his outpatient regimen?

  • A. Continue glyburide at same dose
  • B. Switch glyburide to glipizide
  • C. Add NPH insulin
  • D. Discontinue all diabetes medications

Correct Answer: B. Switch glyburide to glipizide

71. A 72-year-old woman with a history of stroke and heart failure is on warfarin. She is admitted with melena and dizziness. INR is 6.8. Hemoglobin is 7.4 g/dL. She is hemodynamically unstable. Which is the MOST appropriate immediate intervention?

  • A. Hold warfarin only
  • B. Oral vitamin K only
  • C. IV vitamin K plus 4-factor PCC
  • D. Switch to DOAC

Correct Answer: C. IV vitamin K plus 4-factor PCC

72. A 64-year-old man receives a new prescription for lisinopril 10 mg daily for hypertension. He previously experienced angioedema while taking an ACE inhibitor 2 years ago but does not recall the name. BP today is 154/92 mmHg. Which is the MOST appropriate recommendation?

  • A. Start lisinopril 10 mg
  • B. Switch to losartan
  • C. Start combination lisinopril/HCTZ
  • D. Start hydralazine

Correct Answer: B. Switch to losartan

73. A 56-year-old woman with breast cancer is starting doxorubicin-based chemotherapy. Baseline echocardiogram shows LVEF 65 percent. She is anxious about heart damage. Which strategy is MOST appropriate to reduce the risk of cardiotoxicity?

  • A. Give dexrazoxane with therapy
  • B. Add high-dose steroids
  • C. Avoid monitoring LVEF
  • D. Use doxorubicin beyond lifetime maximum

Correct Answer: A. Give dexrazoxane with therapy

74. A 35-year-old man comes to the pharmacy asking for an early refill of oxycodone, stating that his pills were “lost.” Review of the prescription monitoring program shows he received a similar opioid prescription from another prescriber 5 days ago. He appears agitated. Which is the MOST appropriate pharmacist response?

  • A. Refill the prescription to keep him calm
  • B. Refuse to dispense and terminate the relationship
  • C. Engage the patient, discuss concerns, and contact the prescriber
  • D. Call the police immediately

Correct Answer: C. Engage the patient, discuss concerns, and contact the prescriber

75. A 62-year-old woman with type 2 diabetes, hypertension, and hyperlipidemia presents for lipid management. She is taking atorvastatin 80 mg nightly. Recent labs: LDL 78 mg/dL, HDL 42 mg/dL, TG 520 mg/dL. She drinks alcohol socially. Which is the MOST appropriate pharmacologic intervention?

  • A. Add omega-3 fatty acids or fibrate for triglyceride reduction
  • B. Increase atorvastatin
  • C. Discontinue atorvastatin
  • D. Add ezetimibe only

Correct Answer: A. Add omega-3 fatty acids or fibrate for triglyceride reduction

76. A 69-year-old man with newly diagnosed atrial fibrillation is started on warfarin. He is also placed on amiodarone for rhythm control. After 1 week, his INR increases from 1.8 to 3.9. He has no signs of bleeding. Which is the MOST appropriate management strategy?

  • A. Continue warfarin at same dose
  • B. Hold 1 dose and reduce weekly warfarin dose
  • C. Give oral vitamin K 5 mg
  • D. Discontinue amiodarone

Correct Answer: B. Hold 1 dose and reduce weekly warfarin dose

77. A 54-year-old woman with chronic pain is taking a fentanyl transdermal patch. She reports that she cuts the patch in half to “adjust the dose” and sometimes uses a heating pad over the patch for better pain control. Which counseling point is MOST critical?

  • A. Apply patch only to the forearm
  • B. Cutting patches and applying external heat can cause overdose and must be avoided
  • C. Remove the patch before sleeping
  • D. Store patches in the refrigerator

Correct Answer: B. Cutting patches and applying external heat can cause overdose and must be avoided

78. A 47-year-old man with hypertension and chronic gout presents with a recent gout flare treated successfully with NSAIDs. He is currently taking hydrochlorothiazide 25 mg daily and lisinopril 10 mg daily. Uric acid remains elevated. Which antihypertensive should be considered to reduce gout risk?

  • A. Increase hydrochlorothiazide
  • B. Switch hydrochlorothiazide to losartan
  • C. Add propranolol
  • D. Add clonidine

Correct Answer: B. Switch hydrochlorothiazide to losartan

79. A 73-year-old woman is admitted with acute decompensated heart failure. She is receiving IV furosemide. Her potassium is 3.1 mEq/L, and she has frequent premature ventricular contractions on telemetry. She is on lisinopril and metoprolol. Which is the MOST appropriate action?

  • A. Start amiodarone
  • B. Administer IV potassium supplementation
  • C. Stop lisinopril
  • D. Increase metoprolol

Correct Answer: B. Administer IV potassium supplementation

80. A 62-year-old man with depression has been taking sertraline 100 mg daily for 6 weeks with partial improvement. He reports persistent low mood and poor sleep but no suicidal ideation. He has tolerated the medication well. Which is the MOST appropriate next step?

  • A. Discontinue sertraline
  • B. Increase sertraline dose
  • C. Add haloperidol
  • D. Switch immediately to MAOI

Correct Answer: B. Increase sertraline dose

81. A 55-year-old woman with rheumatoid arthritis presents with persistent joint pain and swelling despite weekly methotrexate 25 mg and folic acid. She reports morning stiffness lasting over an hour. Labs: ESR elevated, CRP elevated, SCr normal. She has no active infection, TB test negative last year. She is frustrated with insufficient relief. Which medication should be considered next?

  • A. Add infliximab
  • B. Add hydroxychloroquine only
  • C. Increase folic acid
  • D. Switch methotrexate to prednisone monotherapy

Correct Answer: A. Add infliximab

82. A 67-year-old man with a 40-pack-year smoking history presents with chronic cough, mucus production, and shortness of breath. He uses tiotropium daily and albuterol PRN. He now has 3 exacerbations in 12 months requiring steroids. Vitals stable. CT scan shows emphysema. Which therapy will BEST reduce exacerbation frequency?

  • A. ICS/LABA
  • B. Oral theophylline
  • C. Montelukast
  • D. Nebulized ipratropium only

Correct Answer: A. ICS/LABA

83. A 79-year-old woman is brought by her son for confusion, poor appetite, nausea, and visual disturbances described as “yellow halos.” She takes digoxin for atrial fibrillation, furosemide, and lisinopril. Labs: K⁺ 2.9 mEq/L, digoxin level 3.1 ng/mL. What is the MOST appropriate intervention?

  • A. Increase digoxin dose
  • B. Administer digoxin immune Fab
  • C. Continue current regimen and recheck level
  • D. Add amiodarone

Correct Answer: B. Administer digoxin immune Fab

84. A 42-year-old man presents to the ED with severe chest pain after using cocaine. Vitals: BP 182/110 mmHg, HR 128. ECG shows sinus tachycardia. Troponin pending. Which agent is CONTRAINDICATED for acute blood pressure control in this patient?

  • A. Nitroglycerin
  • B. Lorazepam
  • C. Metoprolol
  • D. Nicardipine

Correct Answer: C. Metoprolol

85. A 63-year-old woman with type 2 diabetes develops a foot ulcer. Culture grows MRSA. She has normal renal function and no allergies. Which oral antibiotic is MOST appropriate?

  • A. Trimethoprim/sulfamethoxazole
  • B. Amoxicillin/clavulanate
  • C. Azithromycin
  • D. Cefdinir

Correct Answer: A. Trimethoprim/sulfamethoxazole

86. A 71-year-old man with atrial fibrillation is starting dabigatran. CrCl is 26 mL/min. He asks if he can open the capsules and sprinkle them on applesauce because he has difficulty swallowing. Which counseling point is MOST accurate?

  • A. Capsule may be opened and mixed with food
  • B. Capsule must be swallowed whole
  • C. Capsule should be crushed for better absorption
  • D. Capsule can be dissolved in water

Correct Answer: B. Capsule must be swallowed whole

87. A 68-year-old woman presents with chronic constipation and abdominal bloating. She uses calcium supplements, iron supplements, and drinks little water. Vitals stable. Exam suggests stool retention. Which intervention is MOST appropriate initially?

  • A. Start polyethylene glycol
  • B. Start linaclotide
  • C. Start lubiprostone
  • D. Start lactulose immediately

Correct Answer: A. Start polyethylene glycol

88. A 60-year-old man presents with increased urinary frequency and urgency. He is on hydrochlorothiazide for hypertension. Digital rectal exam reveals enlarged prostate. PSA normal. Which medication is MOST appropriate for symptomatic improvement?

  • A. Finasteride
  • B. Tamsulosin
  • C. Testosterone
  • D. Oxybutynin

Correct Answer: B. Tamsulosin

89. A 44-year-old man presents with worsening anxiety and panic attacks. He recently started bupropion for depression. Vitals show tachycardia. He denies substance use. Which is the MOST likely cause?

  • A. Bupropion
  • B. Sertraline
  • C. Buspirone
  • D. Propranolol

Correct Answer: A. Bupropion

90. A 52-year-old woman with severe asthma presents for biologic therapy evaluation. She has elevated eosinophils and frequent exacerbations requiring steroids. She uses high-dose ICS/LABA and tiotropium. Which biologic targets eosinophilic asthma?

  • A. Omalizumab
  • B. Mepolizumab
  • C. Adalimumab
  • D. Rituximab

Correct Answer: B. Mepolizumab

91. A 65-year-old man presents with signs of dehydration, tachycardia, and hypotension after taking high-dose loop diuretics. Labs reveal metabolic alkalosis (pH 7.5, HCO₃⁻ elevated), potassium 2.8 mEq/L, chloride 85 mEq/L. Which treatment is MOST appropriate?

  • A. Acetazolamide
  • B. Hypertonic saline
  • C. IV normal saline plus potassium
  • D. IV sodium bicarbonate

Correct Answer: C. IV normal saline plus potassium

92. A 48-year-old woman on long-term prednisone therapy presents with back pain and vertebral compression fracture on imaging. Which medication is MOST appropriate to prevent further bone loss?

  • A. Calcitonin
  • B. Alendronate
  • C. Estrogen
  • D. Calcium alone

Correct Answer: B. Alendronate

93. A 70-year-old man presents with left-sided weakness and slurred speech that began 12 hours ago. CT shows no acute hemorrhage. He is outside the tPA window. Which medication should be initiated immediately?

  • A. IV alteplase
  • B. Aspirin
  • C. Warfarin
  • D. IV heparin

Correct Answer: B. Aspirin

94. A 58-year-old patient with heart failure presents with severe dyspnea. Weight is up 3 kg from baseline. He is on furosemide 40 mg BID at home. In the ED, he receives IV furosemide but has minimal urine output after 2 hours. Which intervention is MOST appropriate?

  • A. Reduce furosemide dose
  • B. Add metolazone
  • C. Start digoxin
  • D. Start verapamil

Correct Answer: B. Add metolazone

95. A 62-year-old man taking lithium for bipolar disorder develops tremors, nausea, vomiting, and confusion. Lithium level is 2.4 mEq/L. He recently started ibuprofen for back pain. SCr increased from 1.2 to 2.0 mg/dL. Which is the MOST likely cause of lithium toxicity?

  • A. Lithium formulation change
  • B. Interaction with ibuprofen causing reduced renal clearance
  • C. High salt intake
  • D. Lithium underdosing

Correct Answer: B. Interaction with ibuprofen causing reduced renal clearance

96. A 44-year-old woman is diagnosed with pulmonary embolism. She has no history of cancer or renal impairment. Weight 72 kg. She prefers oral therapy without injections. Which anticoagulant is MOST appropriate?

  • A. Warfarin only
  • B. Dabigatran
  • C. Apixaban
  • D. Enoxaparin

Correct Answer: C. Apixaban

97. A 70-year-old man on amiodarone presents with fatigue, cold intolerance, and weight gain. Labs show elevated TSH and low free T4. Which intervention is MOST appropriate?

  • A. Increase amiodarone
  • B. Switch amiodarone to digoxin
  • C. Start levothyroxine
  • D. Start liothyronine

Correct Answer: C. Start levothyroxine

98. A 35-year-old woman presents with persistent insomnia. She has already tried good sleep hygiene and melatonin without improvement. She has no substance use history. Which medication is MOST appropriate?

  • A. Diphenhydramine nightly
  • B. Zolpidem
  • C. Alprazolam
  • D. Tramadol

Correct Answer: B. Zolpidem

99. A 59-year-old man is prescribed ciprofloxacin for prostatitis. He takes calcium carbonate daily for reflux and reports taking both together with breakfast. He notes the antibiotic “isn’t working.” Which counseling instruction is MOST appropriate?

  • A. Take ciprofloxacin with calcium for better absorption
  • B. Separate calcium and ciprofloxacin by at least 2 hours
  • C. Increase ciprofloxacin dose
  • D. Stop calcium permanently

Correct Answer: B. Separate calcium and ciprofloxacin by at least 2 hours

100. A 75-year-old woman with hypertension, diabetes, COPD, and mild cognitive impairment presents with dizziness and orthostatic hypotension. She is taking lisinopril, amlodipine, tiotropium, and clonidine. Her standing BP is 92/56 mmHg. Which medication is MOST likely contributing to her symptoms?

  • A. Amlodipine
  • B. Tiotropium
  • C. Clonidine
  • D. Lisinopril

Correct Answer: C. Clonidine

101. A 64-year-old man with type 2 diabetes, coronary artery disease, and stage 3 CKD presents for follow-up. Current medications include metformin 500 mg BID, glipizide 10 mg daily, lisinopril 20 mg daily, atorvastatin 40 mg nightly, and aspirin 81 mg daily. A1C is 8.4%. eGFR is 42 mL/min. He has no history of ketoacidosis. Which diabetes medication would provide additional cardiovascular benefit and is appropriate to add?

  • A. Exenatide
  • B. Empagliflozin
  • C. Acarbose
  • D. Miglitol

Correct Answer: B. Empagliflozin

102. A 70-year-old woman with a history of nonvalvular atrial fibrillation is receiving apixaban 5 mg twice daily. She is 52 kg, age 70 years, and SCr 1.6 mg/dL. She is admitted with worsening renal function (SCr 2.0 mg/dL). According to dosing criteria, which adjustment is MOST appropriate?

  • A. Continue apixaban 5 mg BID
  • B. Reduce apixaban to 2.5 mg BID
  • C. Switch to warfarin
  • D. Stop anticoagulation

Correct Answer: B. Reduce apixaban to 2.5 mg BID

103. A 49-year-old man presents to the ED with severe chest pain, sweating, and nausea. ECG shows ST-segment elevation in the anterior leads. He has no contraindications to thrombolytics. He took sildenafil 3 days ago for erectile dysfunction. Vitals: BP 138/82, HR 98. Which medication is CONTRAINDICATED acutely?

  • A. Aspirin
  • B. Atorvastatin
  • C. Nitroglycerin
  • D. Metoprolol

Correct Answer: C. Nitroglycerin

104. A 59-year-old woman with chronic kidney disease, diabetes, and hypertension presents with hyperkalemia (K⁺ 6.3 mEq/L) on routine labs. EKG shows peaked T waves. Medications include lisinopril, spironolactone, and metformin. She is hemodynamically stable. Which intervention should be given FIRST to stabilize the myocardium?

  • A. IV furosemide
  • B. IV insulin and glucose
  • C. IV calcium gluconate
  • D. Sodium polystyrene sulfonate

Correct Answer: C. IV calcium gluconate

105. A 72-year-old man with long-standing GERD reports new difficulty swallowing solid foods and an unintentional 10-kg weight loss over 3 months. He has been on omeprazole 20 mg daily for years. He denies NSAID use. Which is the MOST appropriate next step in management?

  • A. Increase omeprazole dose
  • B. Add sucralfate
  • C. Schedule upper endoscopy
  • D. Switch to H2 blocker

Correct Answer: C. Schedule upper endoscopy

106. A 63-year-old woman with COPD uses tiotropium daily and salmeterol twice daily. She continues to smoke one pack per day and has had three exacerbations this year requiring steroids. She is anxious about starting “steroids every day.” Vitals: RR 22, SpO₂ 93%. Which inhaled therapy should be added to reduce exacerbations?

  • A. Inhaled corticosteroid
  • B. Nebulized ipratropium only
  • C. Theophylline
  • D. Montelukast

Correct Answer: A. Inhaled corticosteroid

107. A 36-year-old woman presents with a 2-day history of dysuria and urinary frequency. She is not pregnant and has no comorbidities. UA is positive for leukocyte esterase and nitrites. She took trimethoprim/sulfamethoxazole within the last 3 months for a similar infection. Which regimen is MOST appropriate for acute uncomplicated cystitis?

  • A. Nitrofurantoin 100 mg BID for 5 days
  • B. TMP/SMX for 3 days
  • C. Ciprofloxacin for 14 days
  • D. Amoxicillin/clavulanate for 10 days

Correct Answer: A. Nitrofurantoin 100 mg BID for 5 days

108. A 74-year-old man with Parkinson disease has worsening hallucinations. Medications include carbidopa/levodopa, pramipexole, and entacapone. He is oriented but distressed by visual hallucinations. No infection is present. Which adjustment is MOST appropriate to reduce hallucinations while preserving motor function?

  • A. Increase pramipexole
  • B. Reduce or stop pramipexole
  • C. Stop carbidopa/levodopa
  • D. Start haloperidol

Correct Answer: B. Reduce or stop pramipexole

109. A 61-year-old woman presents with acute onset of severe, crushing chest pain radiating to her left arm. ECG shows ST elevation in II, III, aVF. She is taken for emergent PCI and receives aspirin, clopidogrel, and heparin. After successful stent placement, which class of medication must be continued long term in addition to antiplatelets and statin to reduce mortality post-MI?

  • A. ACE inhibitor
  • B. Calcium channel blocker
  • C. Thiazide diuretic
  • D. Alpha-blocker

Correct Answer: A. ACE inhibitor

110. A 52-year-old man presents with severe sore throat, fever, and malaise. Rapid strep test is positive for group A streptococcus. He is allergic to penicillin (anaphylaxis). Which antibiotic is MOST appropriate?

  • A. Amoxicillin
  • B. Azithromycin
  • C. Cefuroxime
  • D. Amoxicillin/clavulanate

Correct Answer: B. Azithromycin

111. A 30-year-old woman with new-onset generalized tonic-clonic seizures is started on valproic acid. She is planning to become pregnant within the next year. Which counseling point is MOST important?

  • A. Valproate is preferred in pregnancy
  • B. Valproate is teratogenic and alternative therapy should be discussed
  • C. No additional precautions are necessary
  • D. Only folic acid is needed and valproate is safe

Correct Answer: B. Valproate is teratogenic and alternative therapy should be discussed

112. A 46-year-old man presents with acute agitation, tremor, sweating, and tachycardia. He started sertraline 2 months ago and was recently started on linezolid for MRSA pneumonia. Vitals: BP 164/96, HR 120, T 38.9°C. He has hyperreflexia and clonus on exam. Which is the MOST likely diagnosis?

  • A. Neuroleptic malignant syndrome
  • B. Serotonin syndrome
  • C. Malignant hyperthermia
  • D. Anticholinergic toxicity

Correct Answer: B. Serotonin syndrome

113. A 70-year-old woman with osteoporosis has been taking alendronate for 8 years. She has had no fractures since starting therapy and T-scores have improved modestly. She tolerates therapy but finds weekly dosing difficult. Which management is MOST appropriate now?

  • A. Continue indefinitely
  • B. Consider a bisphosphonate drug holiday and reassess risk
  • C. Switch to high-dose steroids
  • D. Stop all bone-active therapy permanently

Correct Answer: B. Consider a bisphosphonate drug holiday and reassess risk

114. A 57-year-old man with HIV on antiretroviral therapy (tenofovir, emtricitabine, efavirenz) presents with polyuria and polydipsia. Labs: fasting glucose 168 mg/dL on two occasions. He has no family history of diabetes. Which medication is MOST likely contributing to his hyperglycemia?

  • A. Efavirenz
  • B. Tenofovir
  • C. Emtricitabine
  • D. Lopinavir/ritonavir (if present)

Correct Answer: A. Efavirenz

115. A 60-year-old man with decompensated cirrhosis and esophageal varices presents for prophylaxis. He has had one prior variceal bleed. Vitals are stable. Which medication is recommended for secondary prophylaxis of variceal bleeding?

  • A. Propranolol
  • B. Amlodipine
  • C. Lisinopril
  • D. Furosemide

Correct Answer: A. Propranolol

116. A 49-year-old woman with depression presents with new onset of dry mouth, constipation, blurred vision, and urinary retention after a dose increase of amitriptyline. Which property of amitriptyline is MOST responsible for these effects?

  • A. Alpha-1 blockade
  • B. Anticholinergic activity
  • C. Dopamine blockade
  • D. Histamine-1 antagonism

Correct Answer: B. Anticholinergic activity

117. A 69-year-old man with type 2 diabetes and CKD stage 3 is on metformin 1000 mg BID. His eGFR has declined to 28 mL/min from 38 mL/min. He has no lactic acidosis. Which is the MOST appropriate management regarding metformin?

  • A. Continue metformin at same dose
  • B. Reduce metformin dose by half
  • C. Discontinue metformin
  • D. Switch metformin to glyburide

Correct Answer: C. Discontinue metformin

118. A 44-year-old woman with chronic migraines uses sumatriptan several times per week for acute attacks and OTC ibuprofen frequently. She reports increasing headache frequency and daily baseline headache. Which is the MOST likely cause?

  • A. Medication overuse (rebound) headache
  • B. Brain tumor
  • C. Serotonin syndrome
  • D. Hypertensive crisis

Correct Answer: A. Medication overuse (rebound) headache

119. A 72-year-old man presents to the pharmacy with a prescription for oral potassium chloride 20 mEq BID. He has difficulty swallowing large tablets and asks if he can crush them. The product dispensed is KCl extended-release tablets. What should the pharmacist recommend?

  • A. Crush and mix with applesauce
  • B. Split and crush completely
  • C. Do not crush ER tablets; use a liquid or smaller dosage form
  • D. Reduce dose and crush

Correct Answer: C. Do not crush ER tablets; use a liquid or smaller dosage form

120. A 64-year-old woman with rheumatoid arthritis has been on long-term NSAIDs and low-dose prednisone. She presents with dark stools, fatigue, and dizziness. Labs: Hgb 8.5 g/dL, BUN elevated, SCr normal. Stool guaiac positive. Which is the MOST appropriate next step?

  • A. Start iron supplements only
  • B. Continue NSAIDs but add PPI
  • C. Stop NSAIDs and arrange for endoscopic evaluation
  • D. Reduce prednisone only

Correct Answer: C. Stop NSAIDs and arrange for endoscopic evaluation

121. A 59-year-old man with heart failure with reduced ejection fraction (EF 28%) is taking lisinopril, carvedilol, furosemide, and spironolactone. He remains symptomatic with dyspnea on exertion (NYHA class II–III). BP 110/68, HR 70, K⁺ 4.3 mEq/L, eGFR 55 mL/min. According to guidelines, which additional medication class can further reduce mortality?

  • A. Thiazide diuretic
  • B. ARNI (sacubitril/valsartan)
  • C. Nitrate monotherapy
  • D. Non-DHP CCB

Correct Answer: B. ARNI (sacubitril/valsartan)

122. A 47-year-old woman with asthma has poorly controlled symptoms despite medium-dose ICS/LABA. She has eczema and a history of seasonal allergies. Labs show elevated IgE levels. She has had two oral-steroid-requiring exacerbations in the past year. Which biologic would be most appropriate to consider?

  • A. Omalizumab
  • B. Mepolizumab
  • C. Infliximab
  • D. Adalimumab

Correct Answer: A. Omalizumab

123. A 72-year-old man is prescribed linezolid for MRSA pneumonia. He is also on sertraline for depression. The pharmacist is concerned about a potential interaction. Which serious adverse effect may occur with this combination?

  • A. Neuroleptic malignant syndrome
  • B. Serotonin syndrome
  • C. QT prolongation
  • D. Hemorrhagic stroke

Correct Answer: B. Serotonin syndrome

124. A 33-year-old pregnant woman at 10 weeks gestation presents with nausea and vomiting. She has tried dietary changes without adequate relief. Which medication is considered first-line in pregnancy for nausea and vomiting?

  • A. Ondansetron
  • B. Promethazine
  • C. Doxylamine/pyridoxine
  • D. Metoclopramide

Correct Answer: C. Doxylamine/pyridoxine

125. A 65-year-old man with atrial fibrillation is taking warfarin with a target INR of 2–3. He asks whether he can start an herbal supplement “to boost immunity.” Which supplement is most likely to DECREASE his INR and reduce warfarin effectiveness?

  • A. St. John’s wort
  • B. Ginkgo biloba
  • C. Garlic
  • D. Ginseng

Correct Answer: A. St. John’s wort

126. A 48-year-old woman with bipolar disorder is on carbamazepine. She presents with fever, sore throat, and malaise. Lab tests reveal WBC 1,800/mm³ and ANC 900/mm³. Which is the MOST appropriate action?

  • A. Continue carbamazepine
  • B. Discontinue carbamazepine immediately
  • C. Reduce dose and monitor
  • D. Add filgrastim and continue therapy

Correct Answer: B. Discontinue carbamazepine immediately

127. A 59-year-old man with COPD has been prescribed theophylline. He smokes one pack per day, drinks coffee, and occasionally takes ciprofloxacin. Which factor is MOST likely to INCREASE theophylline levels and risk toxicity?

  • A. Smoking
  • B. Coffee intake
  • C. Concomitant ciprofloxacin
  • D. Age alone

Correct Answer: C. Concomitant ciprofloxacin

128. A 43-year-old woman with obesity and uncontrolled diabetes (A1C 9.0%) is being considered for GLP-1 receptor agonist therapy. She has no personal history of pancreatitis, but her mother had medullary thyroid carcinoma. Which counseling point is MOST important?

  • A. No special risk exists
  • B. GLP-1 agonists have a boxed warning regarding thyroid C-cell tumors in certain patients
  • C. GLP-1 agonists cause hypoglycemia as monotherapy
  • D. They must be combined with insulin

Correct Answer: B. GLP-1 agonists have a boxed warning regarding thyroid C-cell tumors in certain patients

129. A 70-year-old woman with atrial fibrillation and prior stroke is admitted with a hip fracture after a fall. She is on apixaban. Surgery is needed as soon as safely possible. Which parameter primarily guides timing of surgery after last apixaban dose?

  • A. Heart rate
  • B. Liver enzymes
  • C. Renal function and time since last dose
  • D. Blood glucose

Correct Answer: C. Renal function and time since last dose

130. A 29-year-old man presents with acute testicular pain, swelling, and fever. Ultrasound suggests epididymitis. He is sexually active with multiple partners and does not consistently use condoms. Which empiric regimen is MOST appropriate?

  • A. Ceftriaxone plus doxycycline
  • B. Nitrofurantoin
  • C. TMP/SMX
  • D. Metronidazole

Correct Answer: A. Ceftriaxone plus doxycycline

131. A 62-year-old man with chronic atrial fibrillation is transitioning from enoxaparin to warfarin. He has been on both for 4 days, and today his INR is 2.1. He feels well with no bleeding. Which is the MOST appropriate next step?

  • A. Stop enoxaparin today and continue warfarin
  • B. Continue both for at least 5 days and until INR is therapeutic on two consecutive days
  • C. Stop warfarin and continue enoxaparin
  • D. Reduce warfarin and continue enoxaparin indefinitely

Correct Answer: B. Continue both for at least 5 days and until INR is therapeutic on two consecutive days

132. A 55-year-old woman with severe osteoarthritis of the knee has failed acetaminophen and topical NSAIDs. She has a history of peptic ulcer disease. Which analgesic strategy is MOST appropriate?

  • A. Oral nonselective NSAID alone
  • B. COX-2 selective NSAID plus PPI
  • C. Tramadol only
  • D. Aspirin high dose

Correct Answer: B. COX-2 selective NSAID plus PPI

133. A 37-year-old woman presents with acute onset of severe right upper quadrant abdominal pain after a fatty meal, nausea, and vomiting. Ultrasound shows gallstones and gallbladder wall thickening consistent with acute cholecystitis. Which class of antibiotics is MOST appropriate empiric therapy?

  • A. Macrolide monotherapy
  • B. Ceftriaxone plus metronidazole
  • C. Nitrofurantoin
  • D. Linezolid

Correct Answer: B. Ceftriaxone plus metronidazole

134. A 65-year-old woman with chronic atrial fibrillation is stable on warfarin with INR 2.3. She begins a course of TMP/SMX for UTI. Two days later, she returns with epistaxis and INR 4.5. Which factor BEST explains this interaction?

  • A. Increased vitamin K intake
  • B. Induction of warfarin metabolism
  • C. Inhibition of warfarin metabolism and displacement from protein binding
  • D. Decreased warfarin absorption

Correct Answer: C. Inhibition of warfarin metabolism and displacement from protein binding

135. A 72-year-old man with chronic kidney disease is receiving morphine for chronic pain. He develops confusion and myoclonus. Labs show elevated creatinine. Which opioid is PREFERRED in patients with significant renal impairment?

  • A. Morphine
  • B. Codeine
  • C. Hydromorphone or fentanyl
  • D. Meperidine

Correct Answer: C. Hydromorphone or fentanyl

136. A 45-year-old woman presents with persistent insomnia and anxiety. She has been taking zolpidem nightly for 9 months and asks for a dose increase. She feels unable to sleep without it. Which is the MOST appropriate management?

  • A. Continue and increase dose
  • B. Begin gradual taper and consider behavioral therapy
  • C. Switch directly to benzodiazepine
  • D. Stop abruptly

Correct Answer: B. Begin gradual taper and consider behavioral therapy

137. A 59-year-old man with cirrhosis and ascites is on spironolactone and furosemide. He presents with gynecomastia and breast tenderness that he finds intolerable. Which alternative diuretic could be considered to minimize this adverse effect?

  • A. Amiloride
  • B. Eplerenone
  • C. Torsemide
  • D. Triamterene alone

Correct Answer: B. Eplerenone

138. A 68-year-old patient with advanced cancer is receiving high-dose opioids for pain control. He is complaining of severe constipation despite stool softeners. Which is the MOST appropriate additional therapy?

  • A. Add bulk-forming laxative only
  • B. Add stimulant laxative (senna or bisacodyl) ± osmotic
  • C. Reduce opioid dose only
  • D. Add anticholinergic agent

Correct Answer: B. Add stimulant laxative (senna or bisacodyl) ± osmotic

139. A 50-year-old woman with recurrent major depression is responding well to sertraline. After 8 months of therapy with no side effects, she asks if she can stop the medication because she feels “normal.” She has had three prior depressive episodes. Which recommendation is MOST appropriate?

  • A. Stop immediately
  • B. Taper and discontinue now
  • C. Continue long-term maintenance therapy
  • D. Switch to another antidepressant

Correct Answer: C. Continue long-term maintenance therapy

140. A 72-year-old man with HF and CKD is started on an ACE inhibitor. Two weeks later, labs show SCr increased from 1.2 to 1.5 mg/dL and K⁺ from 4.2 to 4.8 mEq/L. He is asymptomatic, BP controlled, and no hyperkalemia symptoms. Which is the MOST appropriate action?

  • A. Stop ACE inhibitor
  • B. Reduce dose slightly and monitor
  • C. Start potassium supplements
  • D. Add spironolactone

Correct Answer: B. Reduce dose slightly and monitor

141. A 60-year-old man presents with new-onset atrial fibrillation and rapid ventricular response. He has heart failure with EF 30%. Which agent is MOST appropriate for acute rate control?

  • A. Verapamil
  • B. Diltiazem
  • C. Metoprolol succinate (IV)
  • D. Digoxin (with or without beta-blocker)

Correct Answer: D. Digoxin (with or without beta-blocker)

142. A 34-year-old woman presents with a history of recurrent genital herpes. She has 6–8 outbreaks per year. She is distressed by the frequency and severity. Which antiviral strategy is MOST appropriate?

  • A. Episodic acyclovir for each outbreak only
  • B. Daily suppressive therapy with acyclovir or valacyclovir
  • C. No antiviral therapy
  • D. Single high-dose famciclovir once per year

Correct Answer: B. Daily suppressive therapy with acyclovir or valacyclovir

143. A 55-year-old man with peptic ulcer disease due to H. pylori completed triple therapy 4 weeks ago. He continues to have mild dyspepsia. Which test is MOST appropriate to confirm eradication?

  • A. Serologic antibody test
  • B. Urea breath test
  • C. Stool guaiac
  • D. Serum gastrin level

Correct Answer: B. Urea breath test

144. A 73-year-old woman with hypertension and heart failure is started on sacubitril/valsartan. She was previously on lisinopril. Which is the MINIMUM recommended washout period between stopping lisinopril and starting sacubitril/valsartan to reduce risk of angioedema?

  • A. 6 hours
  • B. 12 hours
  • C. 24 hours
  • D. 36 hours

Correct Answer: D. 36 hours

145. A 63-year-old man with metastatic prostate cancer receives leuprolide. Shortly after initiation, he experiences worsening bone pain and urinary symptoms. This is best explained by which phenomenon?

  • A. Drug allergy
  • B. Tumor flare due to initial testosterone surge
  • C. Drug failure
  • D. Nephrotoxicity

Correct Answer: B. Tumor flare due to initial testosterone surge

146. A 52-year-old woman with obesity and long-standing poorly controlled diabetes (A1C 10.2%) is started on basal-bolus insulin. She has a history of late meals and frequent snacking. Which adverse effect is MOST important to counsel her about?

  • A. Hyperkalemia
  • B. Hypoglycemia
  • C. Lactic acidosis
  • D. Hyponatremia

Correct Answer: B. Hypoglycemia

147. A 69-year-old man with COPD, hypertension, and diabetes takes multiple medications from several prescribers. During a medication review, the pharmacist identifies duplicate therapy with two different long-acting beta-agonists, both inhaled daily. What is the pharmacist’s BEST immediate action?

  • A. Ignore it since they are similar
  • B. Discontinue both without consulting prescriber
  • C. Contact prescriber to consolidate LABA therapy and prevent overdose
  • D. Reduce the dose of one LABA

Correct Answer: C. Contact prescriber to consolidate LABA therapy and prevent overdose

148. A 58-year-old woman with hypothyroidism is on levothyroxine 100 mcg daily. She starts taking calcium carbonate and ferrous sulfate in the morning. She reports fatigue and weight gain. TSH is elevated, free T4 low. Which counseling point is MOST appropriate?

  • A. Take all medications at the same time with food
  • B. Take levothyroxine on an empty stomach, separate from calcium/iron by at least 4 hours
  • C. Reduce levothyroxine dose
  • D. Stop calcium and iron

Correct Answer: B. Take levothyroxine on an empty stomach, separate from calcium/iron by at least 4 hours

149. A 35-year-old man presents with acute agitation, dilated pupils, dry skin, tachycardia, and urinary retention. He has been taking high doses of diphenhydramine for sleep. Which toxidrome is MOST consistent with his presentation?

  • A. Cholinergic
  • B. Anticholinergic
  • C. Opioid
  • D. Sedative-hypnotic

Correct Answer: B. Anticholinergic

150. A 70-year-old woman with a history of heart failure and atrial fibrillation is taking digoxin. She develops nausea, vomiting, and visual disturbances after starting clarithromycin for pneumonia. Which interaction BEST explains her symptoms?

  • A. Clarithromycin decreases digoxin absorption
  • B. Clarithromycin inhibits P-glycoprotein, increasing digoxin levels
  • C. Clarithromycin induces digoxin metabolism
  • D. Clarithromycin binds digoxin in the gut

Correct Answer: B. Clarithromycin inhibits P-glycoprotein, increasing digoxin levels

151. A 50-year-old man with type 2 diabetes and ASCVD is taking metformin and basal insulin. A new prescription for canagliflozin is written. Which serious adverse effect should be discussed with the patient?

  • A. Thyroid cancer
  • B. Hyperkalemia
  • C. Euglycemic diabetic ketoacidosis and risk of genital infections
  • D. Severe hypoglycemia as monotherapy

Correct Answer: C. Euglycemic diabetic ketoacidosis and risk of genital infections

152. A 43-year-old woman taking combined oral contraceptives is prescribed rifampin for tuberculosis. What is the MOST appropriate counseling point?

  • A. Rifampin has no effect on contraceptives
  • B. Use additional nonhormonal contraception during rifampin therapy
  • C. Stop contraceptives
  • D. Increase contraceptive dose

Correct Answer: B. Use additional nonhormonal contraception during rifampin therapy

153. A 65-year-old man with CKD is prescribed enoxaparin for DVT prophylaxis. His CrCl is 22 mL/min. Which is the MOST appropriate adjustment?

  • A. Use standard prophylactic dose
  • B. Avoid enoxaparin and use unfractionated heparin
  • C. Double the dose
  • D. Use once-weekly dosing

Correct Answer: B. Avoid enoxaparin and use unfractionated heparin

154. A 57-year-old woman with breast cancer is receiving anastrozole. She is postmenopausal and asks about side effects. Which adverse effect is MOST associated with aromatase inhibitors like anastrozole?

  • A. Endometrial cancer
  • B. Osteoporosis and fractures
  • C. Thromboembolism only
  • D. Nephrotoxicity

Correct Answer: B. Osteoporosis and fractures

155. A 60-year-old man with chronic stable angina uses sublingual nitroglycerin. Which counseling statement is CORRECT?

  • A. Store tablets in a weekly pill box
  • B. Take one tablet at onset of chest pain; if pain persists after one dose, call emergency services
  • C. Swallow tablets whole
  • D. Use daily to prevent tolerance

Correct Answer: B. Take one tablet at onset of chest pain; if pain persists after one dose, call emergency services

156. A 48-year-old woman presents with severe nausea and vomiting after starting chemotherapy for breast cancer. She received ondansetron before treatment but still has significant delayed nausea. Which additional agent targets delayed chemotherapy-induced nausea via NK1 receptor antagonism?

  • A. Metoclopramide
  • B. Prochlorperazine
  • C. Aprepitant
  • D. Lorazepam

Correct Answer: C. Aprepitant

157. A 71-year-old man with diabetes, hypertension, and PAD is on aspirin 81 mg and clopidogrel 75 mg daily after stent placement 7 months ago. He has had no recent ischemic events but developed a GI bleed requiring hospitalization. Which antiplatelet strategy is MOST appropriate long-term?

  • A. Continue dual antiplatelet therapy indefinitely
  • B. Stop clopidogrel, continue aspirin alone
  • C. Stop aspirin, continue clopidogrel alone
  • D. Stop all antiplatelet therapy

Correct Answer: B. Stop clopidogrel, continue aspirin alone

158. A 64-year-old woman with chronic low back pain has been taking oxycodone controlled-release for 3 years. She wants to discontinue opioids and asks for help. Which is the MOST appropriate approach?

  • A. Abrupt discontinuation
  • B. Gradual tapering schedule with non-opioid modalities
  • C. Switch to higher potency opioid first
  • D. Add benzodiazepine for withdrawal

Correct Answer: B. Gradual tapering schedule with non-opioid modalities

159. A 38-year-old man presents with acute onset of severe headache, neck stiffness, and photophobia. CT is negative for bleed; lumbar puncture shows elevated opening pressure and neutrophilic pleocytosis. He is started on empiric vancomycin plus ceftriaxone. He is also receiving chronic phenytoin. Which additional medication should be given to reduce neurologic complications of bacterial meningitis?

  • A. Dexamethasone
  • B. Acyclovir
  • C. Metronidazole
  • D. Amphotericin B

Correct Answer: A. Dexamethasone

160. A 55-year-old woman with longstanding uncontrolled hypertension presents with sudden-onset severe headache and BP 220/130 mmHg. CT reveals intracerebral hemorrhage. Which antihypertensive agent is MOST appropriate for acute BP control?

  • A. Oral clonidine
  • B. IV nitroprusside with careful monitoring
  • C. IV labetalol or nicardipine titrated
  • D. Oral ACE inhibitor

Correct Answer: C. IV labetalol or nicardipine titrated

161. A 69-year-old man with COPD is hospitalized for pneumonia. He is on chronic prednisone 10 mg daily. The physician plans a short course of high-dose IV methylprednisolone. Which adverse effect is MOST important to monitor for during this hospitalization?

  • A. Hypoglycemia
  • B. Hyperglycemia and infection risk
  • C. Hypotension
  • D. Bradycardia

Correct Answer: B. Hyperglycemia and infection risk

162. A 51-year-old woman is starting tamoxifen for ER+ breast cancer. She has a history of DVT 5 years ago. Which counseling point is MOST important?

  • A. It cannot cause hot flashes
  • B. It may increase the risk of thromboembolism and requires monitoring of leg swelling, chest pain, or dyspnea
  • C. It decreases risk of endometrial changes
  • D. It has no serious side effects

Correct Answer: B. It may increase the risk of thromboembolism and requires monitoring of leg swelling, chest pain, or dyspnea

163. A 45-year-old man with HIV on protease inhibitor–based therapy develops hyperlipidemia. Which lipid-lowering agent is preferred due to fewer interactions with protease inhibitors?

  • A. Simvastatin
  • B. Lovastatin
  • C. Atorvastatin high dose
  • D. Pravastatin

Correct Answer: D. Pravastatin

164. A 72-year-old woman with heart failure and atrial fibrillation presents with fatigue and anorexia. She is on digoxin, furosemide, and lisinopril. Labs show K⁺ 3.0 mEq/L, digoxin level at upper therapeutic range, SCr normal. Which factor most increases her risk of digoxin toxicity?

  • A. Hypokalemia
  • B. Hyperkalemia
  • C. Low digoxin level
  • D. High blood pressure

Correct Answer: A. Hypokalemia

165. A 30-year-old man with new-onset Crohn disease is started on infliximab. Before therapy, he received screening labs. Which infection must be ruled out due to boxed warning of reactivation with TNF-alpha inhibitors?

  • A. HIV
  • B. Hepatitis A
  • C. Tuberculosis
  • D. HSV-2

Correct Answer: C. Tuberculosis

166. A 67-year-old man presents for evaluation of insomnia. He has COPD and still smokes. He asks for a “strong sleeping pill” and mentions a friend’s alprazolam. Which is the MOST appropriate response?

  • A. Benzodiazepines are first-line in COPD
  • B. Avoid benzodiazepines; consider non-pharmacologic therapy and safer agents
  • C. Prescribe alprazolam
  • D. Recommend alcohol at bedtime

Correct Answer: B. Avoid benzodiazepines; consider non-pharmacologic therapy and safer agents

167. A 59-year-old man with hypertension, diabetes, and CKD is on lisinopril, amlodipine, and HCTZ. BP is 150/88. He has albuminuria and eGFR 38 mL/min. Which additional antihypertensive may be particularly beneficial for both BP and proteinuria?

  • A. Add spironolactone cautiously
  • B. Switch ACE inhibitor to ARB plus ACE together
  • C. Add non-DHP CCB like verapamil (carefully)
  • D. Add alpha-blocker only

Correct Answer: A. Add spironolactone cautiously

168. A 33-year-old woman with a history of opioid use disorder is maintained on buprenorphine/naloxone. She is scheduled for moderate surgical procedures and expresses concern about pain control. Which is the MOST appropriate strategy?

  • A. Stop buprenorphine immediately and give no opioids
  • B. Coordinate with anesthesia to use multimodal analgesia and possibly higher opioid doses due to partial agonist blockade
  • C. Switch to methadone for 1 day
  • D. Use NSAIDs only

Correct Answer: B. Coordinate with anesthesia to use multimodal analgesia and possibly higher opioid doses due to partial agonist blockade

169. A 76-year-old woman on multiple medications presents with confusion and falls. Her regimen includes oxybutynin, diphenhydramine, amitriptyline, and lisinopril. Which concept from geriatric pharmacotherapy explains the need to minimize these agents?

  • A. Beers Criteria and anticholinergic burden
  • B. Narrow therapeutic index
  • C. High first-pass metabolism
  • D. Enzyme induction

Correct Answer: A. Beers Criteria and anticholinergic burden

170. A 58-year-old man with peptic ulcer disease is prescribed omeprazole long-term. He asks about possible long-term risks. Which risk has been associated with prolonged PPI use?

  • A. Severe hypokalemia
  • B. Increased risk of C. difficile infection and fractures
  • C. Hyperthyroidism
  • D. Parkinson disease

Correct Answer: B. Increased risk of C. difficile infection and fractures

171. A 62-year-old woman with recurrent venous thromboembolism is starting lifelong warfarin therapy. She is counseled on diet. Which instruction is MOST appropriate?

  • A. Avoid all leafy green vegetables
  • B. Keep vitamin K intake consistent from week to week
  • C. Eat more vitamin K to reverse warfarin
  • D. Take vitamin K supplements daily

Correct Answer: B. Keep vitamin K intake consistent from week to week

172. A 49-year-old man presents with persistent cough and shortness of breath. He has been on amiodarone 400 mg daily for 8 months for atrial fibrillation. Chest X-ray shows diffuse interstitial changes. Which adverse effect is MOST likely?

  • A. Amiodarone-induced pulmonary toxicity
  • B. COPD exacerbation
  • C. Pneumothorax
  • D. Pulmonary embolism

Correct Answer: A. Amiodarone-induced pulmonary toxicity

173. A 40-year-old woman with history of depression presents after an overdose of acetaminophen. Serum acetaminophen level is above the nomogram treatment line. Which antidote is indicated?

  • A. N-acetylcysteine
  • B. Flumazenil
  • C. Naloxone
  • D. Atropine

Correct Answer: A. N-acetylcysteine

174. A 72-year-old man with diabetes and prior MI is on aspirin, ACE inhibitor, beta-blocker, statin, and metformin. He asks if he should take over-the-counter ibuprofen daily for joint pain. Which is the MOST appropriate advice?

  • A. Safe to take daily, no concerns
  • B. Chronic NSAID use may increase CV and renal risk; use lowest dose for shortest time or consider alternatives
  • C. Replace aspirin with ibuprofen
  • D. Stop ACE inhibitor if using ibuprofen

Correct Answer: B. Chronic NSAID use may increase CV and renal risk; use lowest dose for shortest time or consider alternatives

175. A 60-year-old woman with hypothyroidism is well controlled on levothyroxine. She is diagnosed with atrial fibrillation and started on amiodarone. Six months later, she has fatigue and weight gain. Labs show low TSH and high free T4. Which is the MOST likely thyroid effect?

  • A. Overt hypothyroidism
  • B. Hyperthyroidism induced by amiodarone
  • C. Normal thyroid function
  • D. Sick euthyroid syndrome

Correct Answer: B. Hyperthyroidism induced by amiodarone

176. A 67-year-old man with a history of GI bleed is starting low-dose aspirin after stent placement. To reduce risk of recurrent GI bleeding, which co-therapy is MOST appropriate?

  • A. H2 blocker low dose
  • B. Proton pump inhibitor
  • C. Sucralfate only
  • D. Antacid only

Correct Answer: B. Proton pump inhibitor

177. A 25-year-old woman with epilepsy is considering pregnancy. She is taking valproic acid and lamotrigine. Which medication poses the highest teratogenic risk and should be avoided if possible?

  • A. Lamotrigine
  • B. Valproic acid
  • C. Both are low risk
  • D. Neither is teratogenic

Correct Answer: B. Valproic acid

178. A 54-year-old man with severe depression has failed two trials of SSRIs. His psychiatrist recommends ECT. The patient is fearful of “shock treatment.” Which is the MOST appropriate counseling message?

  • A. ECT is obsolete and rarely used
  • B. ECT can be effective and is performed under anesthesia with monitoring
  • C. ECT is unsafe and causes brain damage
  • D. ECT must be repeated daily for life

Correct Answer: B. ECT can be effective and is performed under anesthesia with monitoring

179. A 70-year-old woman is starting denosumab for osteoporosis. She has normal renal function but poor dentition and has not seen a dentist in years. Which serious adverse effect should be discussed?

  • A. Hyperkalemia
  • B. Osteonecrosis of the jaw
  • C. Hepatotoxicity
  • D. Cataracts

Correct Answer: B. Osteonecrosis of the jaw

180. A 61-year-old man with uncontrolled hypertension despite triple therapy is being evaluated for secondary causes. Labs reveal hypokalemia and metabolic alkalosis with high aldosterone and low renin. Which diagnosis is MOST likely?

  • A. Cushing disease
  • B. Primary hyperaldosteronism (Conn syndrome)
  • C. SIADH
  • D. Addison disease

Correct Answer: B. Primary hyperaldosteronism (Conn syndrome)

181. A 45-year-old woman presents with severe acute asthma exacerbation despite multiple albuterol nebulizations. She received systemic steroids 1 hour ago and is on oxygen. She continues to wheeze with RR 32 and SpO₂ 90%. Which IV medication can be considered as adjunct therapy in severe refractory asthma?

  • A. IV magnesium sulfate
  • B. IV propranolol
  • C. IV furosemide
  • D. IV digoxin

Correct Answer: A. IV magnesium sulfate

182. A 73-year-old man with benign prostatic hyperplasia is started on finasteride. He also takes tamsulosin. Which key counseling point should be given regarding finasteride?

  • A. It works immediately
  • B. It may take several months to improve symptoms and can reduce PSA levels
  • C. It increases testosterone significantly
  • D. It treats acute urinary retention

Correct Answer: B. It may take several months to improve symptoms and can reduce PSA levels

183. A 53-year-old woman with uncontrolled GERD is being evaluated for Barrett esophagus. Endoscopy confirms Barrett changes. She is on omeprazole 20 mg daily. Which is the MOST appropriate modification?

  • A. Discontinue PPI
  • B. Increase PPI dose and continue long-term
  • C. Switch to H2 blocker only
  • D. Use antacids only

Correct Answer: B. Increase PPI dose and continue long-term

184. A 60-year-old man with COPD and chronic respiratory failure is hospitalized for an exacerbation. He receives high-dose IV steroids and broad-spectrum antibiotics. Two weeks later, he develops watery diarrhea, abdominal pain, and fever. Stool test is positive for C. difficile toxins. Which regimen is FIRST-line for severe C. difficile infection (current recommendations)?

  • A. Oral metronidazole
  • B. Oral vancomycin
  • C. IV vancomycin
  • D. Oral ciprofloxacin

Correct Answer: B. Oral vancomycin

185. A 28-year-old woman presents with acute onset of dysuria and vaginal discharge. She is sexually active with multiple partners. Pelvic exam shows cervical motion tenderness and purulent discharge. Pregnancy test is negative. Which diagnosis is MOST likely?

  • A. Uncomplicated cystitis
  • B. Pelvic inflammatory disease
  • C. Vulvovaginal candidiasis
  • D. Atrophic vaginitis

Correct Answer: B. Pelvic inflammatory disease

186. A 68-year-old man with diabetes and coronary artery disease is on insulin glargine, metformin, aspirin, lisinopril, and atorvastatin. He presents with blood glucose 45 mg/dL, confusion, and diaphoresis. After correction, which is the MOST important counseling point?

  • A. Skip insulin on days he feels well
  • B. Eat consistent meals and monitor for hypoglycemia symptoms
  • C. Stop metformin
  • D. Stop aspirin

Correct Answer: B. Eat consistent meals and monitor for hypoglycemia symptoms

187. A 76-year-old woman with dementia, hypertension, and osteoarthritis is started on donepezil. Her caregiver reports increased urinary incontinence since initiation. This is likely due to which pharmacologic action of donepezil?

  • A. Anticholinergic effects
  • B. Beta-blockade
  • C. Cholinesterase inhibition increasing cholinergic activity
  • D. Alpha-1 antagonism

Correct Answer: C. Cholinesterase inhibition increasing cholinergic activity

188. A 64-year-old man is being treated for tuberculosis with isoniazid, rifampin, pyrazinamide, and ethambutol. After 2 months, ethambutol and pyrazinamide are discontinued. He now complains of orange discoloration of urine and tears. Which drug is responsible?

  • A. Isoniazid
  • B. Rifampin
  • C. Pyrazinamide
  • D. Ethambutol

Correct Answer: B. Rifampin

189. A 52-year-old woman presents with new-onset atrial fibrillation and rapid ventricular response (HR 150). She has normal EF and no structural heart disease. Which medication is MOST appropriate for acute rate control?

  • A. IV diltiazem
  • B. IV flecainide
  • C. Digoxin alone
  • D. Amiodarone oral

Correct Answer: A. IV diltiazem

190. A 69-year-old man with type 2 diabetes, CKD, and peripheral neuropathy presents with tingling and burning pain in his feet. He is on metformin, insulin, lisinopril, and atorvastatin. Which medication is MOST appropriate for neuropathic pain?

  • A. Ibuprofen
  • B. Gabapentin
  • C. Tramadol only
  • D. Acetaminophen

Correct Answer: B. Gabapentin

191. A 71-year-old woman presents with progressive shortness of breath and dry cough. She has rheumatoid arthritis and has been on methotrexate for several years. Chest imaging shows diffuse interstitial infiltrates. Which adverse effect is MOST likely?

  • A. Methotrexate-induced pulmonary toxicity
  • B. COPD
  • C. Pulmonary embolism
  • D. Pneumothorax

Correct Answer: A. Methotrexate-induced pulmonary toxicity

192. A 63-year-old man with hyperthyroidism is treated with methimazole. He presents with sore throat and fever. CBC shows WBC 1,500/mm³ and ANC 500/mm³. Which is the MOST appropriate management?

  • A. Continue methimazole
  • B. Discontinue methimazole immediately and evaluate for agranulocytosis
  • C. Add G-CSF but continue methimazole
  • D. Reduce dose and observe

Correct Answer: B. Discontinue methimazole immediately and evaluate for agranulocytosis

193. A 54-year-old woman with chronic kidney disease is prescribed a phosphate binder. She has a history of hypercalcemia. Which phosphate binder is MOST appropriate?

  • A. Calcium carbonate
  • B. Calcium acetate
  • C. Sevelamer
  • D. High-dose vitamin D

Correct Answer: C. Sevelamer

194. A 67-year-old man with insomnia and depression is prescribed trazodone at bedtime. Which side effect should be discussed, especially at higher doses?

  • A. Priapism
  • B. Hypoglycemia
  • C. Bradycardia
  • D. Seizures

Correct Answer: A. Priapism

195. A 59-year-old man is started on allopurinol for chronic gout prevention. He is also on azathioprine for a kidney transplant. Which important interaction should the pharmacist recognize?

  • A. Allopurinol induces azathioprine metabolism, reducing effect
  • B. Allopurinol inhibits xanthine oxidase, increasing azathioprine toxicity
  • C. No interaction
  • D. Azathioprine reduces allopurinol absorption

Correct Answer: B. Allopurinol inhibits xanthine oxidase, increasing azathioprine toxicity

196. A 42-year-old man with schizophrenia is being treated with clozapine. His ANC is monitored regularly. Today his ANC is 1,100/mm³ (low but not critically low). He has no symptoms. Which is the MOST appropriate action?

  • A. Continue and recheck ANC per REMS guidelines with increased monitoring
  • B. Discontinue clozapine immediately
  • C. Double clozapine dose
  • D. Stop monitoring

Correct Answer: A. Continue and recheck ANC per REMS guidelines with increased monitoring

197. A 60-year-old woman with major depression is switched from phenelzine (an MAOI) to sertraline. How long should she wait after stopping phenelzine before starting sertraline to avoid serotonin syndrome?

  • A. 24 hours
  • B. 3 days
  • C. 7 days
  • D. 14 days

Correct Answer: D. 14 days

198. A 73-year-old man is admitted with pneumonia and is started on IV vancomycin and cefepime. He develops redness, pruritus, and hypotension during rapid vancomycin infusion. Which reaction is MOST likely?

  • A. Anaphylaxis
  • B. Red man syndrome (histamine-mediated response to rapid infusion)
  • C. SJS
  • D. Serum sickness

Correct Answer: B. Red man syndrome (histamine-mediated response to rapid infusion)

199. A 52-year-old woman on chronic opioid therapy for cancer pain is started on an SSRI for depression. She reports that her pain is “somewhat improved” but she is sleeping better and coping better. Which principle does this illustrate?

  • A. SSRIs directly block pain pathways
  • B. Treating depression can improve pain perception and function
  • C. Opioid doses are too low
  • D. SSRIs are analgesics

Correct Answer: B. Treating depression can improve pain perception and function

200. A 66-year-old man with hypertension and diabetes presents with a new prescription for chlorthalidone. He is already taking lisinopril and amlodipine. Which electrolyte abnormality is MOST commonly associated with thiazide diuretics like chlorthalidone?

  • A. Hyperkalemia
  • B. Hyponatremia and hypokalemia
  • C. Hypercalcemia
  • D. Hypocalcemia

Correct Answer: B. Hyponatremia and hypokalemia

📥 Download Full 200-Question PDF

Get the complete question bank with answers and explanations for offline study.

Download PDF Now